You are on page 1of 27

1. There are at least 5 factors that affect the process of labor and birth.

These are commonly referred to


as the 5 Ps: passenger, passageway, powers, position of mother and psyche. The size of the fetal
head has a major effect on the birth process. During a vaginal exam of a client in labor, the nurse
palpates the fetus' larger, diamond shaped fontanelle toward the anterior portion of the client’s pelvis.
Which of the following statements best describes this situation?
A. The client can expect a brief and intense labor, which potential for lacerations
B. The client is at risk for uterine rupture and needs constant monitoring

C. The client may need interventions to ease back pain and change fetal position

D. The fetus will be delivered using forceps or vacuum extractor

Rationale:
 The fetal position is occiput posterior, a position that commonly produces intense back pain during
labor.
 Most of the time, the fetus rotates during labor to occiput anterior position.
 Positioning the client on her side can facilitate this rotation.
 An occiput posterior position would most likely result in prolonged labor
 POSTERIOR - PROLONGED.
occiput posterior alone doesn’t create a risk of uterine rupture.
 The fetus won’t be delivered with forceps / vacuum only if it’s presenting part DOESNT rotate and
descend spontaneously.
(Lippincott's, p.101)

2. Presentation refers to the part of the fetus that enters the pelvic inlet first. Lie is the relationship of the
long axis of the fetus with the long axis of the mother. Attitude is the relation of fetal body parts to one
another. Which of the following is most favorable for birth?
A. Vertex
B. Transverse lie
C. Frank breech presentation
D. Posterior position of the head

Rationale:
Refers to the fetal body part that enter the pelvis first. Three different types of presentations can occur:

-Cephalic (vertex)- Head first, the most favorable and most common (96%) presentation.
-Breech - Feet or buttocks first
-Shoulder - Transverse lie
(Maternity and Pediatric Nursing, S. Scott, T. Kyle, p.366)

3. The passageway in labor and delivery of the fetus include the following EXCEPT
A. Distensibility of lower uterine segment
B. Cervical dilatation and effacement
C. Distensibility of vaginal canal and introitus
D. Flexibility of the pelvis

Rationale:
The pelvis is a bony structure that is part of the passageway but is not flexible. The lower uterine
segment including the cervix as well as the vaginal canal and introitus are all part of the passageway in
the delivery of the fetus.

4. The passage refers to the route a fetus must travel from the uterus through the cervix and vagina to the
external perineum. The cervix and vagina are contained inside the pelvis, a fetus must also pass
through the bony pelvic ring. A pregnant woman asks the nurse what is the implication f having
gynecoid pelvis? The nurse's best response should be:
A. That gynecoid pelvis has a narrow pubic arch
B. That gynecoid pelvis is the most favorable for labor and birth
C. That gynecoid pelvis is a wide pelvis, but has a short diameter
D. That this type of pelvis is not favorable for a normal labor and vaginal delivery and a woman who
has this type of pelvis will need a cesarean section

Rationale: A gynecoid pelvis is a normal female pelvis and is the most favorable for successful labor
and birth. An android pelvis (resembling a male pelvis) would be unfavorable for labor because of the
narrow pelvic planes. An anthropoid pelvis has an outlet that is adequate, with a normal or moderately
narrow pubic arch. A platypelloid pelvis (flat pelvis) has a wide transverse diameter, but the
anteroposterior diameter is short, making the outlet inadequate.

(Saunders Comprehensive Review for the NCLEX-RN Examination, 5thed. By Silvestri, p 258-259)
5. Engagement is the term used to indicate that the largest transverse diameter of the presenting part has
passed through the maternal pelvic brim or inlet. After completing a second vaginal examination of a
client in labor, the nurse determines that the fetus is in the right occiput anterior position and at –1
station. Based on these findings, the nurse knows that the fetal presenting part is:

A. 1 cm below sacral promontory


B. 1 cm above ischial spines
C. 1 cm below ischial spines
D. 1 cm above ischial tuberosities

Rationale:
1cm above ischial spines

 The relationship of the baby’s presenting part to the ischial spines is called fetal station. When the
baby’s head reach zero station, delivery is imminent as the largest part of the baby’s head has entered
the bony pelvis.
 In total, there are seven fetal stations: -3, -2, -1, 0, +1, +2 and +3. The description of fetal engagement
in each station can be easily remembered by the acronym FISHING.
 In -3, the baby is still floating far from the pelvis. In -2, the baby is going in the right direction. In -1, the
baby is settling in entering the pelvic cavity and in 0 the baby is already halfway to delivery. In +1 the
baby is inching out and in +2 the baby is nearly there. In +3 station, the baby’s head is already
crowning out.
(Maternal and Child Health Nursing, A. Pillitteri)

6. To determine whether a primigravid client in labor with a fetus in the Left Occipito Anterior (LOA)
position is completely dilated, the nurse performs vaginal examination. During the examination, the
nurse would expect to palpate which of the following cranial sutures?
A. Frontal
B. Coronal
C. Sagittal
D. Lambdoid

Rationale:
 Sagittal suture: separates parietal bones and extends longitudinally back to front
 Frontal suture: between two frontal bones, continuation of the sagittal suture
 Coronal suture: as a crown; separates frontal and parietal bones
 Lambdoid suture: separates occipital bone from two parietal bones
(Mosby’s, p.365)

7. The onset of true labor cannot be ascribed to a single cause. The earliest sign preceding labor is:
A. Bloody show
B. Lightening
C. Return of urinary frequency
D. Cervical ripening

Rationale:
 Signs preceding labor
 Lightening ( dropping into pelvic area)
 Bloody show ( brownish or blood tinged cervical mucus)
 Surge of energy
 Weight loss of 0.5 to 1.5 kg in weight
(Maternity and Pediatric Nursing, S. Scott, T. Kyle, p.360)

8. The first stage of labor has been divided into 3 phases: latent, active and transitional. A client comes to
the emergency room stating that she thinks that she is in labor. The nurse should tell the client that
labor has probably begun when:
A. Her contractions are 2 minutes apart.
B. She has back pain and a bloody discharge.
C. She experiences abdominal pain and frequent urination.
D. Her contractions are irregular.

Rationale:

Bloody show, she should not wait until the contractions are every 2 minutes or until she has bloody
discharge. She should also be told to report to the hospital if she experiences rupture of membranes or
extreme bleeding. The client should be advised to come to the labor and delivery unit when the contractions
are every 5 minutes and consistent.
(Maternity and Pediatric Nursing, S. Scott, T. Kyle, p.360)
9. Nurse Luisa is assessing a primigravid client in active labor who did not receive any analgesia or
anesthesia. Her findings include: complete cervical effacement, dilatation of 8cm, fetus at station 0.
Which of the following behaviors would the nurse anticipate the client will exhibit during this phase?
A. Relief
B. Excitement
C. Loss of control
D. Ambivalence
Rationale:
The client is in the transitional phase. During this phase, a woman may experience intense discomfort, so
strong that it is accompanied by nausea and vomiting. Because of the intensity and duration of the
contractions, a woman may also experience a feeling of loss of control, anxiety, panic, or irritability. Sensations
may be so intense it may seem as though labor has taken charge of her.
(Maternal and Child health Nursing, 6thed. By Pilliterri, p. 361)

10. A client diagnosed with preterm labor at 28 weeks gestation. Later, she comes to the emergency dept.
saying "I think I’m in labor" The nurse would expect her physical exam to show which condition?
A. painful contractions with no cervical dilation
B. regular uterine contractions with cervical dilation
C. irregular uterine contractions with no cervical dilation
D. irregular uterine contractions with cervical effacement

Rationale:
Regular uterine contraction (every 10minutes or more) along with cervical dilatation before 36 weeks’ gestation
or rupture of fluids indicates preterm labor. Uterine contractions without cervical change don’t indicate preterm
labor.
(Springhouse Review, p. 224)

11. A woman with a term, uncomplicated pregnancy comes into L&D in early labor saying that she thinks
her water broke. Which action should the nurse take?
A. Prepare the woman for delivery
B. Note color, amount and odor of fluid
C. Immediately contact the doctor
D. Collect sample of fluid for microbial analysis

Rationale:
 Noting color, amount and odor of the fluid as well as the time of the rupture, will help guide the nurse in
her next action.
 There's no need to call the doctor immediately or prep the client for delivery if the fluid is clear and
delivery isn’t imminent.
 ROM isn’t unusual in early stages of labor.
Fluid collection for microbe analysis isn’t routine and there’s no concern for infection/maternal fever.
(Maternity and Pediatric Nursing, S. Scott, T. Kyle, p.1412)

12. A 16 year old teenage pregnant client was admitted to the labor and delivery unit. Her assessment
reveals the following: cervix 2 cm dilated, 50% effaced; intact membranes, contractions every 7
minutes. She is in latent phase of the first stage of labor. Which of the following would the nurse
recommend after the client is admitted?
A. Walking around in the hallway.
B. Resting in the right lateral recumbent position
C. Sitting in a comfortable chair for a period of time.
D. Lying in the left lateral recumbent position

Rationale:
Most authorities suggest that a woman in an early stage of labor should be allowed to walk if she wishes as
long as no complications are present. Birthing centers and single-room maternity units allow women
considerable latitude without much supervision at this stage of labor. Gravity and walking can assist the
process of labor in some clients. If the client becomes tired, she can rest in bed in the left lateral recumbent
position or sit in a comfortable chair. Resting in the left lateral recumbent position improves circulation to the
fetus.

Situation: Mrs. Alvarez G3P2 comes to the labor and delivery unit and reports ruptured amniotic membranes
and contractions that occur every 3 minutes lasting 50-60 seconds. Upon assessment, the fetus is said to be in
LOA position.

12. The nurse’s priority action should be to:


A. Check the FHR
B. Check the vaginal discharge with nitrazine paper
C. Perform vaginal examination
D. Call the physician
Rationale:

When membranes rupture, the priority focus is on assessing fetal heart rate first to identify a deceleration,
which might indicate cord compression secondary to cord prolapse. A vaginal exam may be done later to
evaluate for continued progression of labor. The physician should be notified, but this is not a priority at this
time. Changing the linen saver pad would be appropriate once the fetal status is determined and the physician
has been notified.
(Maternity and Pediatric Nursing, S. Scott, T. Kyle, p.363)

13. The fetal heart rate is checked following rupture of the bag of waters in order to:
A. Check if the fetus is suffering from head compression
B. Determine if cord compression followed the rupture
C. Determine if there is utero-placental insufficiency
D. Check if fetal presenting part has adequately descended following the rupture

Rationale:
After the rupture of the bag of waters, the cord may also go with the water because of the pressure of the
rupture and flow. If the cord goes out of the cervical opening, before the head is delivered (cephalic
presentation), the head can compress on the cord causing fetal distress. Fetal distress can be detected
through the fetal heart tone. Thus, it is essential do check the FHB right after rupture of bag to ensure that the
cord is not being compressed by the fetal head.

14. The nurse established an IV line, and then connects Mrs. Alvarez to an electronic fetal monitor. The fetal
monitoring strip shows FHR deceleration occurring about 30 sec after each contraction begins; the FHR
returns to baseline after the contraction is over. This type of deceleration is caused by:
A. Fetal head compression
B. Umbilical cord compression
C. Utero-placental insufficiency
D. Cardiac anomalies

Rationale:
 The fetal monitoring strip show late deceleration caused by utero-placental insufficiency- inadequate
fetal oxygenation resulting from decreased blood flow during uterine contraction. Utero-placental
insufficiency may result from maternal hypotension, tetanic contractions, postmaturity, abruption
placentae, or PIH. (NCLEX-RN Review Made Incredibly Easy, p.548)
 Late decelerations are caused by decreased oxygen saturation as a result of a problem with the
uteroplacental unit. Fetal head compression manifests in early decelerations. Fetal cardiac
abnormalities are not associated with late decelerations, and cord compression typically manifests
in variable decelerations.
(Maternity and Pediatric Nursing, S. Scott, T. Kyle, p.364)

15. With this type of deceleration, the nurse’s first action should be to:
a. Do nothing, this is a normal occurrence
b. Call the physician
c. Position the patient on her left side
d. Continue monitoring the FHR

Rationale:
In this type of deceleration, change the woman’s position from supine to lateral (to relieve pressure on the vena
cava and supply more blood to the uterus and fetus).
(Maternal and Child health Nursing, 6thed. By Pilliterri, p. 380)

16. If the external fetal monitor shows a pattern of variable decelerations in fetal heart rate. What should the
nurse do first?
A. Change the client’s position.
B. Prepare for emergency cesarean section.
C. Check for placenta previa.
D. Administer oxygen.

Rationale:
Variable decelerations in fetal heart rate are an ominous sign, indicating compression of the umbilical cord.
Changing the client’s position from supine to side-lying may immediately correct the problem. An emergency
cesarean section is necessary only if other measures, such as changing position and amnio-infusion with
sterile saline, prove unsuccessful. Administering oxygen may be helpful, but the priority is to change the
woman’s position and relieve cord compression.
17. The following are signs and symptoms of fetal distress EXCEPT:
A. Fetal heart rate (FHR) decreased during a contraction and persists even after the uterine contraction
ends
B. The FHR is less than 120 bpm or over 160 bpm
C. The pre-contraction FHR is 130 bpm, FHR during contraction is 118 bpm and FHR after uterine contraction
is 126 bpm
D. FHR is 160 bpm, weak and irregular

Rationale:
The normal range of FHR is 120-160 bpm, strong and regular. During a contraction, the FHR usually goes
down but must return to its pre-contraction rate after the contraction ends

18. The fetal heart beat should be monitored every 15 minutes during the 2nd stage of labor. The characteristic
of a normal fetal heart rate is
A. The heart rate will decelerate during a contraction and then go back to its pre-contraction rate after the
contraction
B. The heart rate will accelerate during a contraction and remain slightly above the pre-contraction rate at the
end of the contraction
C. The rate should not be affected by the uterine contraction.
D. The heart rate will decelerate at the middle of a contraction and remain so for about a minute after the
contraction

Rationale:
The normal fetal heart rate will decelerate (go down) slightly during a contraction because of the compression
on the fetal head. However, the heart rate should go back to the pre-contraction rate as soon as the
contraction is over since the compression on the head has also ended.

19. Uterine contractions are the primary powers that act involuntarily to expel the fetus and the placenta from
the uterus. It begins with a slow increment, gradually reaches an acme and then diminishes rapidly. A uterine
contraction is described in terms of the following charactertics: frequency, duration, intenstiy and interval. The
contraction of Larisse started at 6:03pm and lasted until 6:05pm and the next contraction occurred at 6:10pm.
Based on the information, what is the frequency of her contractions?
a. 2 minutes
b. 5 minutes
c. 7 minutes
d. 9 minutes

Rationale:
The assessment is done by palpation; duration, frequency, intensity, and resting tone must be assessed.
The duration of contractions is measured in seconds; the frequency is measured in minutes.
The intensity of contractions usually is described as mild, moderate, or strong.
The resting tone usually is characterized as soft or relaxed.
(Maternal Child Nursing Care, S. Perry, p.411)

20. The proper technique to monitor the intensity of a uterine contraction is


A. Place the palm of the hands on the abdomen and time the contraction
B. Place the finger tips lightly on the suprapubic area and time the contraction
C. Put the tip of the fingers lightly on the fundal area and try to indent the abdominal wall at the height of the
contraction
D. Put the palm of the hands on the fundal area and feel the contraction at the fundal area

Rationale:
In monitoring the intensity of the contraction the best place is to place the fingertips at the fundal area. The
fundus is the contractile part of the uterus and the fingertips are more sensitive than the palm of the hand.

21. To monitor the frequency of the uterine contraction during labor, the right technique is to time the
contraction.
A. From the beginning of one contraction to the end of the same contraction
B. From the beginning of one contraction to the beginning of the next contraction
C. From the end of one contraction to the beginning of the next contraction
D. From the deceleration of one contraction to the acme of the next contraction

Rationale:
Duration is the time from the beginning to the end of the single contraction. Frequency is the time from the
beginning of one contraction to the beginning of the next contraction. Acme refers to the peak of a contraction.
Intensity refers to the strength of the contraction.
22. Nurse Andrea documented that Larisse’s contractions are moderate. This means that upon palpation of the
fundus, it felt like touching:
a. The tip of the nose
b. The chin
c. The forehead
d. The cheekbone

Rationale:
A contraction that feels like the chin typically represents a moderate contraction. A contraction described as
feeling like the tip of the nose indicates a mild contraction. A strong contraction feels like the forehead.
(The Labor Progress Handbook, p.83)

23. A nurse is admitting a pregnant client to the labor room and attaches an external electronic fetal monitor to
the client’s abdomen. What is the initial action of the nurse after attachment of the electronic fetal monitor?
A. Identify the types of accelerations.
B. Assess the baseline fetal heart rate.
C. Determine the intensity of the contractions.
D. Determine the frequency of the contractions.

Rationale: Assessing the baseline fetal heart rate is important so that abnormal variations of the baseline
rate can be identified if they occur. The intensity of contractions is assessed by an internal fetal monitor,
not an external fetal monitor. Options A and D are important to assess, but not as the first priority. Fetal
heart rate is evaluated by assessing baseline and periodic changes. Periodic changes occur in response to
the intermittent stress of uterine contractions and the baseline beat-to-beat variability of the fetal heart rate.
(Saunders Comprehensive Review for the NCLEX-RN Examination, 5thed. By Silvestri, p 308-311)

24. Nurse Andrea also plans to check on Larisse’s bladder. The rationale for this action is that the urinary
bladder should not be allowed to become distended; primarily since a full bladder tends to?
a. Increase the risk of urinary incontinence.
b. Predispose to a prolapsed of the umbilical; cord.
c. Interfere with observation of the uterine contraction.
d. Prevent the descent of the fetus in the birth canal.

Rationale:
A full bladder contributes to the client’s discomfort and impedes fetal descent, possibly prolonging labor.
(Maternal and Child health Nursing, 6th ed. By Pilliterri, p. 386)

25. The cervical dilatation taken at 8:00 A.M. in a G1P0 patient was 6 cm. A repeat I.E. done at 10 A.M.
showed that cervical dilation was 7 cm. The correct interpretation of this result is:
A. Labor is progressing as expected
B. The latent phase of Stage 1 is prolonged
C. The active phase of Stage 1 is protracted
D. The duration of labor is normal

The active phase of Stage I starts from 4cm cervical dilatation and is expected that the uterus will dilate by 1cm
every hour. Since the time lapsed is already 2 hours, the dilatation is expected to be already 8 cm. Hence, the
active phase is protracted.

26. When evaluating the client in labor, nurse Andrea must know that primiparas normally are not taken to the
delivery room until?
a. The cervix is dilated 6 to 8 cm.
b. Contractions are 3 to 5 minutes apart.
c. The intensity of the contraction decrease.
d. There is complete cervical dilatation.

Rationale:
Preparing the Place of Birth: For a multipara, convert a birthing room into a birth room by opening the sterile
packs of supplies on waiting tables when the cervix has dilated to 9 to 10 cm. For a primipara, this can be
delayed until the head has crowned to the size of a quarter or half-dollar (full dilatation and descent).
(Maternal and Childhealth Nursing, 6th ed. By Pilliterri, p. 385)

27. A client who’s admitted to labor and delivery has the following assessment findings: gravida 2 para 1,
estimated 40 weeks’ gestation, contractions 2 minutes apart, lasting 45 seconds, vertex +4 station. Which
of the following would be the priority at this time?
A. Placing the client in bed to begin fetal monitoring.
B. Preparing for immediate delivery.
C. Checking for ruptured membranes.
D. Providing comfort measures.
This question requires an understanding of station as part of the intrapartal assessment process. Based on the
client’s assessment findings, this client is ready for delivery, which is the nurse’s top priority. Placing the client
in bed, checking for ruptured membranes, and providing comfort measures could be done, but the priority here
is immediate delivery.

28. A client is in the second stage of labor. This stage begins with complete cervical dilatation and ends with
expulsion of the fetus. During this stage, how frequently should the nurse assess the FHR?

A. After every contraction


B. Every 5-15 minutes
C. Every 30 minutes
D. Every 60 minutes

During the second stage of labor, the nurse should assess the strength, frequency, and duration of
contraction every 15 minutes. If maternal or fetal problems are detected, more frequent monitoring is
necessary. An interval of 30 to 60 minutes between assessments is too long because of variations in
the length and duration of patient’s labor.

29. On December 7, 2009, the Department of Health issued an Administrative Order implementing the ENC
protocol with the goal of rapidly reducing the number of newborn deaths in the Philippines. With
international standards integrated in Intrapartal care the program has now evolved into the EINC program
under the guidance of the W.H.O. The unnecessary interventions during labor and delivery, which do not
improve the health of mother and child, are eliminated, this include which of the following?
1. Use of fundal pressure to help the mother in the expulsion of her fetus
2. Application of IV therapy to hydrate women due to food and drink restrictions
3. Early amniotomy and oxytocin augmentation in order to prevent operative delivery
4. Shaving the pubic hair of women in labor as hygienic practice to minimize infection
5. Use of enema to reduce the risk of infections and shorten the duration of labor
6. Restricted intake of food and fluid during active labor for possible risk of aspirating gastric
contents due to anesthesia
a. 5, 4, and 6 c. All of these 6
b. 2, 1, and 3 d. None of these

30. Freedom of maternal ambulation and choice of position throughout labor are likely to be beneficial for the
woman in labor. Which of the following statements regarding maternal positions during labor and delivery is
not true?
A. In an upright position, gravity enhances contraction cycle and fetal descent.
B. The hands-and-knees position is the ideal position when the presenting part is in a posterior
position
C. Semi-recumbent is associated with less frequent, but more intense contractions.
D. Lateral position reduces risk of perineal trauma.

Rationale:
Advantages of Semi-recumbent
 At least 30 degrees and pillow under one hip to prevent supine hypotension
 The greater the angle, the more gravity will promote fetal descent and progress of contractions
 Position is convenient for rendering care measures and external fetal monitoring
(Nursing Care of the Family during Labor and Birth)

31. The physician plans to artificially rupture the membranes, a procedure known as amniotomy of a
multiparous client admitted at 41 weeks of gestation for labor induction. After the said procedure, which of
the following nursing interventions is the priority?
A. Assess fetal heart rate for 1 full minute
B. Assess the client’s vital signs
C. Assess the client’s contraction pattern
D. Document the color of the amniotic fluid.

Rationale:
The first nursing action after the membranes are ruptured is to check the FHR. Compression of the cord could
occur after rupture leading to fetal hypoxia as reflected in an alteration in FHR pattern, characteristically
variable decelerations. The same initial action should follow artificial rupture of the membranes (amniotomy).

32. After teaching a pregnant woman who is in labor about the purpose of the episiotomy, which of the
following purposes stated by the client would indicate to the nurse that the teaching was effective?
A. Shortens the second stage of labor
B. Enlarges the pelvic inlet
C. Prevents perineal edema
D. Ensures quick placenta delivery
Explanation
An episiotomy serves several purposes. It shortens the second stage of labor, substitutes a clean surgical
incision for a tear, and decreases undue stretching of perineal muscles. An episiotomy helps prevent tearing of
the rectum but it does not necessarily relieves pressure on the rectum. Tearing may still occur.

33. The third stage of labor lasts from birth of baby until delivery of placenta. Placental separation is indicated
by which of the following signs, except:
A. A firmly contracted uterus
B. Lengthening of the umbilical cord
C. Change in uterus from globular ovoid to discoid shape
D. Sudden gush of dark blood

Rationale:
Signs of placental separation include lengthening of the umbilical cord, a sudden gush of dark blood from the
introitus (vagina), a firmly contracted uterus, and the uterus changing from a discoid (like a disk) to a globular
(like a globe) shape. The client may experience vaginal fullness, but not severe uterine cramping. (Foundations
and Adult Health Nursing, p.811)

34. What are the important considerations that the nurse must remember after the placenta is delivered?
1. Check if the placenta is complete including the membranes
2. Check if the cord is long enough for the baby
3. Check if the umbilical cord has 3 blood vessels
4. Check if the cord has a meaty portion and a shiny portion
A. 1 and 3
B. 2 and 4
C. 1, 3, and 4
D. 2 and 3

Rationale:
The nurse after delivering the placenta must ensure that all the cotyledons and the membranes of the placenta
are complete. Also, the nurse must check if the umbilical cord is normal which means it contains the 3 blood
vessels, 2 veins and 1 artery.

35. Nurse Mary, is assisting the newly hired Nurse Mark in the labor and delivery of a multiparous woman.
After the delivery of the placenta, Nurse Mark checks if the placenta is complete. What would he do next?
A. Check the institution's policy on proper placental disposal
B. Ask the patient if they would need the placenta
C. Donate the placenta to be used for stem cell transplantation
D. Ask Nurse Mary on where to dispose the placenta

Rationale:
For many women the placenta has continuing importance. For this reason, women may ask if they can take it
home with them. In several Asian and Native American cultures, women bury the placenta to ensure that the
child will continue to be healthy. In some parts of China, the placenta is cooked and eaten to ensure the
continued health of the mother. Ask parents whether saving the placenta is important to them before it is
destroyed. Be certain when supplying placentas to women to take home with them that you respect standard
infection precautions and hospital policy. In major health centers, women may be asked to donate their
newborn’s placenta so blood can be removed and banked to be available for bone marrow or stem cell
transplantation (Scott, 2007). Placental membranes can be salvaged to be used as temporary coverings for
burns.
(Maternal and Child health Nursing, 6th ed. By Pilliterri, p.362)

POSTPARTUM
36. Few changes in vital signs are seen in the postpartum period. Nurse Suzanne is assessing several women
who just gave birth. Which of the following women has normal expected changes in the postpartum period?
A. Marissa, whose temperature is 37.8C 6 hours after delivery
B. Claire, whose PR is 128 8 hours after delivery
C. Kim, whose BP 80/60 5 hours after delivery
D. Alice, whose RR is 11bpm 4 hours after delivery.

Rationale:
 Temperature oral or axillary a slightly elevated temperature less than 100.4 degrees is normal. If
temperature rises above 100.4 she is considered febrile. If elevated temperature lasts be thinking
infection.
 Pulse rate is slightly lower than normal. 60-70bpm. By the end of the first week pulse rate will return to
normal. Rapid and thready pulse is a sign of hemorrhage.
 Compare pressure to pre-pregnancy pressure. A drop in pressure can indicate bleeding. Elevation
above 140 mm Hg systolic or 90 mm Hg diastolic may indicate post-partal pregnancy-induced
hypertension.
37. While assessing a primipara during the immediate postpartum period, the nurse in charge plans to use
both hands to assess the client’s fundus to:
A. Prevent uterine inversion
B. Promote uterine involution
C .Hasten the puerperium period
D. Determine the size of the fundus

Rationale:
Using both hands to assess the fundus is useful for the prevention of uterine inversion.

38. Upon assessment, the nurse got the following findings: 2 perineal pads highly saturated with blood within 2
hours post-partum, PR= 80 bpm, fundus soft and boundaries not well defined. The appropriate nursing
diagnosis is:
A. Normal blood loss
B. Blood volume deficiency
C. Inadequate tissue perfusion related to hemorrhage
D. Hemorrhage secondary to uterine atony

All the signs in the stem of the question are signs of hemorrhage. If the fundus is soft and boundaries not well
defined, the cause of the hemorrhage could be uterine atony.

39. If the labor period lasts only for 3 hours, the nurse should suspect that the following conditions may occur:
1. Laceration of cervix
2. Laceration of perineum
3. Cranial hematoma in the fetus
4. Fetal anoxia

A.1 & 2
B.2 & 4
C.2, 3, 4
D.1, 2, 3, 4

All the above conditions can occur following a precipitate labor and delivery of the fetus because there
was little time for the baby to adapt to the passageway. If the presentation is cephalic, the fetal head
serves as the main part of the fetus that pushes through the birth canal which can lead to cranial
hematoma, and possible compression of cord may occur which can lead to less blood and oxygen to
the fetus (hypoxia). Likewise the maternal passageway (cervix, vaginal canal and perineum) did not
have enough time to stretch which can lead to laceration.

40. The return of the uterus to a nonpregnant state after birth is known as involution. This process begins
immediately after expulsion of the placenta with contraction of the uterine smooth muscles. Eight hours
postpartum, a client’s fundus is 3 cm above the umbilicus and displaced to the right. The statement by the
client to the nurse that is most significant would be:
a. “I’ve been so thirsty the past few hours.”
b. “I’ve changed my pads once since I got to my room.”
c. “I’ve had a lot of contractions, especially when nursing.”
d. “I’ve been up to the bathroom but can’t seem to urinate.”

Rationale:
Involution is assessed by measuring fundal height. A full bladder impedes uterine contraction by pushing
upward on the uterus and displacing it.
(Introductory Maternity Nursing, p.272)

41. Postpartum uterine discharge, commonly called lochia is initially bright red. While making a home visit to a
postpartum client on day10, the nurse would anticipate that the client’s discharge would be which of the
following colors?
A. Dark Red
B. Pink
C. Brown
D. White

Philippine laws have been recommending exclusive breastfeeding for the first 6 months of life and that
breastfeeding be continued beyond 6 months complemented with solid food.

42. Laceration refers to tears in the perineum, vagina, or cervix from the stretching of tissues during delivery.
Lacerations are classified as first, second, third, or fourth degree. A client is 3 days postpartum. She states
that she hasn’t had a bowel movement since before delivery and is experiencing discomfort. She has had a
fourth-degree laceration. The nurse knows that the best remedy is:
A. a suppository.
B. an enema to alleviate gas pains quickly.
C. stool softeners and fluids.
D. pain medication for the discomfort.

Rationale:
Degrees of Laceration
 First-degree laceration involves the vaginal mucosa and the skin of the perineum and fourchette.
 Second-degree laceration involves the vagina, perineal skin, fascia, levator ani muscle, and perineal
body.
 Third-degree laceration involves the entire perineum and the external anal sphincter.
 Fourth-degree laceration involves the entire perineum and rectal sphincter and portions of the rectal
mucosa.

A client with a fourth-degree laceration is at risk for dehiscence. Stool softeners and fluid will gently promote
stool evacuation. Suppositories and an enema would be too harsh, and pain medications would slow down
peristalsis of the intestines, slowing evacuation.
(NCLEX-RN Review made Incredibly Easy, 5th edition, p. 553)

BREASTFEEDING

43. Roxanne, a primiparous client who is beginning to breastfeed her neonate asks the nurse about colostrum.
It is a clear yellowish fluid that is more concentrated than mature milk. When instructing the client, the
nurse would explain that colostrum provides her neonate with:
A. Vitamin K, which the baby lacks
B. More fat than breastmilk
C. Passive immunity from maternal antibodies
D. Delayed meconium passage

Rationale:
Colostrum and breast milk may provide immunity. Passive immunity is acquired by developing fetus when it
receives maternal antibodies in utero, or by baby when it receives maternal antibodies contained in colostrum
and breast milk.
(Foundations of Nursing Practice: Fundamentals of Holistic Care, p.344)

44. Roxanne is also curious about the nutritional content of colostrum. The nurse explains that colostrum
contains the following :
A. High protein, high fat, high sugar content
B. High protein, low fat, low sugar content
C. Low protein, low fat, high sugar content
D. Low protein, low fat, low sugar content

Rationale:
Colostrum, a thin, watery, yellow fluid composed of protein, sugar, fat, water, minerals, vitamins, and
maternal antibodies, is secreted by the acinar breast cells starting in the fourth month of pregnancy. For the
first 3 or 4 days after birth, colostrum production continues. Because it is high in protein and fairly low in
sugar and fat, colostrum is easy to digest and capable of providing adequate nutrition for a newborn until it
is replaced by transitional breast milk on the second to fourth day. True or mature breast milk is produced
by the 10th day.
(Maternal and Child health Nursing, 6th ed. By Pilliterri, p.491 )

45. Roxanne tells the nurse that she is aware of a “let down sensation” in her breasts when she feeds her
baby. This is also referred to as milk ejection reflex. The nurse explains that the letdown sensation is
stimulated by:
A. Estrogen
B. Prolactin
C. Oxytocin
D. Progesterone

Rationale:
Contraction of the milk ducts and let-down reflex occur under the stimulation of oxytocin released by the
posterior pituitary gland. (Foundations of Maternal-Newborn and Women's Health Nursing, p446)

46. The client who is breast-feeding asks the nurse if she should supplement breast feeding with formula
feeding. The nurse bases the response on which of the following?
a. Formula feeding should be avoided to prevent interfering with the breast milk supply
b. Primarily, water supplements should be used to prevent jaundice
c. Formula supplements can provide nutrients not found in breast milk
d. More vigorous sucking is needed for a bottle-feeding, so supplements should be avoided.
Rationale:
Bottle supplements tend to cause a decrease in the breast milk supply and demand for breast-feeding and
should be avoided. Once in a while if the client is tired, a bottle supplement may be given to the neonate by
another caregiver. Bottle supplements are not appropriate to prevent jaundice, although if neonatal bilirubin
level is excessive, some pediatricians recommend temporary discontinuation of breast-feeding, while others
recommend increasing the frequency of breast-feeding. Breast-feeding is considered the best nutritional
source for infants. Although formula supplements should be avoided, neonates suck less vigorously on a bottle
than on the breast.
(Maternal-Child Nursing Care Enhanced, p.496)

47. On the third postpartum day, a client who is breast-feeding calls the clinic complaining of hot, hard, aching
breast. This is known as engorgement, which is characterized by painful overfilling of the breasts. Nursing
intervention should include advising her to:
A. Limit the number of times the baby nurses
B. Air dry the breast for 20 minutes after nursing
C. Use ice packs continuously throughout the day
D. Apply warm, moist cloths to both breast before nursing

Rationale:
If an infant cannot grasp a nipple to suck strongly because of engorgement, warm packs applied to
both breasts or standing under a warm shower for a few minutes before feeding, combined with massage to
begin milk flow, often promotes breast softness so an infant can suck.
(Maternal and Child health Nursing, 6th ed. By Pilliterri, p. 498)

48. Suzette, given birth to her first child, plans to breastfeed her neonate. She asks the nurse about the
interventions she could do for having inverted nipple. The nurse is correct if she responds which of the
following:
A. You can practice nipple rolling so that your nipples will be more protuberant
B. You may need to wear a nipple cup to make your nipples more protuberant
C. You may switch to bottle feeding instead.
D. Both A and B

Rationale:
The occasional woman who has inverted nipples may need to wear a nipple cup (a plastic shell) to help her
nipples become more protuberant. Physical preparation such as nipple rolling, advised in the past as a way
of making a woman’s nipples more protuberant, is not necessary because few women have inverted or non
protuberant nipples. In addition, oxytocin, which is released by this maneuver, could lead to preterm labor
(Maternal and Child health Nursing, 6th ed. By Pilliterri, p. 494)

50. There are 4 basic positions for breastfeeding: cradle, modified cradle, football hold and side-lying.
Important health education topics for breastfeeding include proper positioning and latching of the baby.
A 2 weeks postpartum patient informs the nurse that her nipples have become sore and cracked from
the feedings. Which of the following would the nurse instruct the client to do?
A. Feed the baby less often for the next 7 days.
B. Use mild soap and water to wash the area to prevent an infection.
C. Wipe off any lanolin creams from the nipple before each feeding.
D. Position the baby with the entire areola in the baby’s mouth.
Rationale:
For the breastfeeding mother, engorgement is often the result of vascular congestion and milk stasis, primarily
caused by the infant not fully emptying the mother's breasts at each feeding. Cracking of the nipple could lead
to infection. Improper positioning may lead to nipple tenderness or pain. Inadequate secretion of prolactin
causes a decrease in the production of milk.

SITUATION 1: Alex on her 12 weeks gestation with her third baby sough consultation to a nurses clinic for
observation. She has a history of spontaneous abortion and is spotting.

51. She told the nurse she had minimal vaginal bleeding, without passage of placenta with embryonic sac
and slight uterine cramping. On examination, the physician on duty determines that her cervix is closed.
The nurse would think that the client is exhibiting signs of:
A. Septic abortion
B. Missed abortion
C. Complete abortion
D. Incomplete abortion
Core Competency: Health Education– Analysis
Rationale:
Missed Abortion – the fetus dies in the utero but is not expelled because the cervix maybe closed and a
woman may have had symptoms of threatened miscarriage (painless vaginal bleeding). There’s no increase in
fundal height.
 Complete Abortion - the entire products of conception (fetus, membranes, and placenta) are expelled
spontaneously with minimal bleeding.
 Incomplete Abortion – part of conceptus (usually the fetus) is expelled, but membrane or placenta is
retained in the uterus. Bleeding can occur because part of the conceptus is retained in the uterus
because the uterus cannot contract effectively in this condition.
 Septic Abortion – an abortion that is complicated by infection. Symptoms are fever, crampy abdominal
pain, uterus feels tender to palpate. Occurs in women who have tried to self-abort or were aborted
illegally using nonsterile instrument.
(PILLITERI 5TH ED., pg, 405-407)

52. A threatened abortion is suspected at the 10 weeks pregnant client, Becky. Nurse Sarah instructs the
client regarding management of care. Which statement made by the Becky indicates aneed for further
instructions?
A. “I will watch for the evidence of the passage of tissue.”
B. “I will maintain strict bedrest throughout the remainder of the pregnancy.”
C. “I will count the number of perineal pads used on a daily basis and note the amount and color of blood
on the pad.”
D. “I will avoid sexual intercourse until the bleeding has stopped, and for 2 weeks following the last
evidence of bleeding.”

Rationale:
Strict bedrest throughout the remainder of the pregnancy is not required for a threatened abortion. The client is
advised to curtail sexual activities until bleeding has ceased and for 2 weeks after the last evidence of bleeding
or as recommended by the physician or other health care provider. The client is instructed to count the number
of perineal pads used daily and to note the quantity and color of blood on the pad. The client also should watch
for the evidence of the passage of tissue.
(Saunders Comprehensive Review for the NCLEX-RN Examination, 5th ed. By Silvestri, p 294,296)

53. The client at 12 weeks comes to the clinic with signs/symptoms of abdominal cramping and moderate
vaginal bleeding and ultrasound results of absent fetal heart tones. On internal examination, the
physician determines that her cervix is closed. Which nursing intervention is most appropriate to the
client?
A. Prepare the client for curettage
B. Place the client in Trendelenburg position
C. Prepare the client for cerclage
D. Prepare the client in a side lying position
Core Competency: Health education– Application

Rationale:
A patient with missed abortion should undergo DILATATION AND CURETTAGE. If the pregnancy is over
14 weeks, labor may be induced by a prostaglandin suppository or misoprostol (cytotec) to dilate the cervix,
followed by oxytocin stimulation or administration of mifepristone. Also monitor for infection and disseminated
intravascular coagulation (DIC) may develop if the dead fetus remains too long in the utero.
 Cervical cerclage, is for patient with Habitual abortion
 Side lying position and Trendelenburg position, is for patient with threatened abortion.
(PILLITERI 5TH ED., pg, 405-407)

54. While preparing for the Dilatation and Curretage (D&C), the student nurse asked the Nurse Paul which
of the following instruments will not be used in the procedure?
A. Tenaculum
B. Curette
C. Speculum
D. None of the above

Rationale:
In D&C, the uterus is then scraped clean with a curette, removing the zygote and trophoblast cells with the
uterine lining.
The speculum is used to spread the vaginal wall and tenaculum is used to hold the cervix open during the
procedure.

(Maternal and Childhealth Nursing, 6th ed. By Pilliterri, p. 141)


(www.hopkinsmedicine.org)
Concept: Abortion
 Types
 Description and etiology
 Signs and symptoms
 Management

Ectopic pregnancy is the implantation of the products of conception in a site other than the uterine cavity with
great majority in the ampulla of the fallopian tube. The greatest risks of developing ectopic pregnancy are
pelvic inflammatory disease, contraceptive device and history of previous ectopic pregnancy or fallopian tube
surgery. Diagnostic test includes ultrasound, vital signs monitoring and culdocenteris. Surgical treatment is
salpingectomy or salpingostomy.
Situation 2: A client arrives in the emergency department with amenorrhea for two months. She was previously
treated for a Chlamydial infection.
55. The nurse would assess the client suspected of ectopic pregnancy. Which assessment findings by the
nurse do not indicate the presence of this condition?
A. Unilateral pain radiating to the shoulder
B. Cervical tenderness on internal examination
C. Profuse vaginal bleeding
D. Abdominal examination finding of a bluish navel
Core competency: Safe and Quality Care – Analysis

Rationale:
A patient with Ectopic Pregnancy exhibits the following symptoms: scant vaginal bleeding, sharp
stabbing pain one of her lower abdominal quadrants (unilateral), rigid and tender abdomen, may develop a
bluish tinge in umbilicus (Cullen’s sign), pain in her shoulders from blood in the peritoneal cavity causing
irritation to the phrenic-nerve, and a tender mass is usually palpable in Dougla’s cul-de-sac on vaginal
examination.
(PILLITERI 5TH ED., pg, 408 - 409)

56. The physician ordered to prepare a client suspected of ectopic pregnancy with a negative culdocentesis
result. Which statement by the client indicates understanding of the management for her condition?
A. “I will be maintained in a left lateral position.”
B. “An abdominal scrub will be performed in preparation for CS.”
C. “My hemoglobin and hematocrit will be periodically monitored.”
D. “I will not be asked to sign an informed consent.”
E. Sterile gloves, vaginal speculum, lubricant and lamp
Core Competency: Safe and Quality Care – Analysis

Rationale:
Culdocentesis is a procedure in which peritoneal fluid is obtained from the cul de sac of a female
patient. It involves the introduction of a spinal needle through the vaginal wall into the peritoneal space of the
pouch of Douglas.
 Normal culdocentesis result: A normal culdocentesis result in the absence of pathology should yield
only 2-4 mL of clear to straw-colored peritoneal fluid.
 A negative culdocentesis is indicated by the presence of clear fluid.
 A positive result refers to the free flow of non clotting blood.
 If no fluid is obtained, the test is considered non diagnostic.
Website: http://emedicine.medscape.com/article/83097-overview#a9

57. Elena, is diagnosed to have ectopic pregnancy. She is to receive medical intervention rather than a
surgical interruption. Which of the following intramuscular medications would the physician prescribe?
A. Decadron (desamethasone)
B. Amethopterin (methotrexate)
C. Pergonal (menotropins)
D. Prometrium (progesterone)
Rationale:
Methotrexate is an antineoplastic agent. Methotrexate is a folic acid antagonist that interferes with DNA
synthesis and cell multiplication. The conceptus is a ball of rapidly multiplying cells. Methotrexate interferes
with that multiplication, killing the conceptus and,therefore, precluding the need for the client to undergo
surgery.
Decadron is a steroid. Pergonal is an infertility medication. Progesterone injections are administered to clients
who have a history of preterm labor.
(Maternal and Newborn Success: A Course Review Applying Critical Thinking to Test Taking, by De Sevo, p.
244, 263)
Concept: Ectopic pregnancy
 Description and etiology
 Clinical manifestations
 Diagnostic tests
 Management
Hydatidiform mole is an abnormal growth of trophoblastic tissue including the placenta and chorion. Two
distinct types of hydatidiform mole are complete and partial mole. A complete mole develops from an empty
ovum that contains no maternal genetic material. A partial H mole may have an abnormal embry o that usually
spontaneous abort in the first trimester. Suction curettage or hysterectomy may be desired surgical
management. The most serious complication of H. mole during its acute phase is hemorrhage

Situation 3: A prenatal client at 14 weeks gestation with no prenatal are present to the labor and delivery unit
with nausea and vomiting and a severe headache. The client has elevated blood pressure. .
58. When assessing the client, which of the following signs and symptoms by the nurse would confirm the
presence of a hydatidiform mole?
A. Painless, heavy vaginal bleeding
B. Unusual uterine enlargement
C. Decreased fetal heart tones
D. BP 80/60
Core competency: Safe and Quality Care – Analysis

Rationale:
In patient with Hytatidiform mole, the sign and symptoms associated with this condtion are:
˃ Dark brown to bright red vaginal bleeding during the first trimester
˃ Severe nausea and vomiting
˃ Sometimes vaginal passage of grape-like cysts
˃ Rarely pelvic pressure or pain
˃ Rapid uterine growth — the uterus is too large for the stage of pregnancy
˃ High blood pressure
˃ No fetal heart sounds are heard because there is no viable fetus
˃ Positive pregnancy test – hcg level 1 to 2 million IU), produced by by the trophoblast cells
Website:http://www.mayoclinic.org/diseases-conditions/molar-pregnancy/basics/symptoms/con-20034413
Book: (PILLITERI 5TH ED., pg, 411)

59. There are several risk factors for having H-mole. Nurse Mira is teaching a group of student nurses
about it. Which of the following would most likely develop the disease.
A. High socioeconomic status
B. Primigrvida
C. 20 to 30 years of age
D. Prior molar gestation

Rationale:
A previous molar gestation increases a woman's risk for developing a subsequent molar gestation by four to
five times. Adolescents and women ages 40 years and older are at increased risk for molar pregnancies.
Multigrvidas, especially woman with a prior pregnancy loss and women with lower socioeconomic status are at
an increased risk for this problem.
(NCLEX-RN Questions & Answers made Incredibly Easy!, 6th ed. By Lisko, p. 1668)

60. The pregnant client with molar pregnancy was treated with suction curettage. The nurse recognizes
that additional discharge teaching is required when the client states which of the following discharge
health instructions?
A. “I am so sad that I lost this baby.”
B. “I may need to have chemotherapy after this hospitalization.”
C. “I will need to see the doctor yearly for follow-up.”
D. “I will be using contraception for the next two years.”
Core competency: Health Education – Application

Rationale:
Nursing consideration after Suction Curettage
a) A woman should be instructed to use contraceptive methods such as oral contraceptive agents for 2
years.
b) A woman may experience the same feeling of loss after the evacuation, because after all, she
believes she was pregnant.
c) Also a woman can seek a doctor for chemotherapy since malignancy can maybe develop.
(PILLITERI 5TH ED., pg, 410-412)
Concept: Hydatidiform mole
 Description
 Types
 Clinical manifestations
 Management
 Health teaching
 Complications
Incompetent cervix is described as a painless effacement and dilatation of the cervix that is not associated with
contractions and usually occurs in the second trimester resulting in spontaneous abortion or very preterm birth.
Maternal DES exposure or congenital uterine anomalies may be associated with incompetent cervix. Other
possible contributing factors are cervical inflammation or previous cervical trauma. Management is through a
cerclage.

Situation 4. A client who experienced mild vaginal bleeding and was diagnosed with incompetent cervix has
had a McDonald cerclage procedure done at 18 weeks in the current pregnancy.
61. The client calls the clinic at 37 weeks because she is having irregular contractions every 5 to 7 minutes.
Which response by the nurse is most appropriate?
A. “Wait and come in when the contractions are closer and harder.”
B. “Come to the hospital to have your cerclage removed before your baby is born.”
C. You sound like you are worried about this baby. It must be frightening you.”
D. “You need to have a cesarean birth with the cerclage in place.”
Core competency: Safe and Quality Care – Application

Rationale:
After the loss of one child due to premature cervical dilatation, a surgical operation termed cervical
cerclage can be performed to prevent this condition from happening again. With these procedures, the sutures
are then removed at weeks 37 to 38 of pregnancy so the fetus can be born vaginally.
(PILLITERI 5TH ED., pg, 412)

Concept: Incompetent cervix


 Description and etiology
 Clinical manifestations
 Management

The placenta in previa is abnormally implanted near to or over the internal cervical os. As the cervix softens
and begins to efface and dilate, placental sinuses are opened causing progressive hemorrhages.
Transabdominal ultrasound confirms suspicion of placenta previa. The degree to which the placenta covers the
os leads to different classifications such as total, partial, marginal and low lying placenta previa. Predisposing
factors include multifetal pregnancy, multiparity, and scarring in the upper third of the uterus.

Situation 5: Mrs. Madlangpipol, a G4P3 is admitted to the prenatal clinic at 8 months pregnantwith complaint of
painless vaginal bleeding. Diagnosis is incomplete placenta previa.
62. Upon admission, the nursing measure to which the nurse should perform INITIALLY is to:
A. Set–up for emergency low cervical cesarean section
B. Elevate the foot of the bed, check cervical dilatation, check the vital signs
C. Assess the amount and character of the vaginal bleeding
D. Check the fetal heart tones and set up for oxygen therapy
Core competency: Safe and Quality Care – Application
Rationale:
Nursing measures for placenta previa:
1) Immediate nursing care
- Place the woman immediately on bed rest in a sidelying position.
- Be sure to assess
 Duration of pregnancy
 Time the bleeding began
 Woman’s estimation of the amount of blood.
 Color of the blood
2) Prior episode of bleeding during the pregnancy never attempt pelvic or rectal examination because any
agitation of the cervix when there is a placenta previa may initiate massive hemorrhage.
3) Continue to assess blood pressure
4) Keep IV line and monitor urine output frequent
5) Blood replacement if necessary
6) Must have double set up (Operating room or a fully equipped birthing room) if hemorrhage does occur
after vaginal examinations.
7) Have oxygen equipment available in case the fetal heart sounds indicate fetal distress such as
bradycardia or tachycardia, late deceleration or variable decelerations.
(PILLITERI 5TH ED., pg, 413-414)

63. A 35 year old multigravid client who is at 32 weeks gestation is admitted to the obstetric unit for
observation. The admission diagnosis is total placenta previa. Based on the client’s clinical
presentation, which admission information should the nurse obtain first?
A. Pregnancy and prior delivery history
B. Blood pressure and pulse rate
C. General health and drug history
D. Height and weight

Core Competency: Safe and Quality Care - Application


Rationale:
In a patient who’s have placenta previa, is better to assess her vital sign to determine whether
symptoms of shock are presents.

(PILLITERI 5TH ED., pg, 412-14)

64. Basha, a pregnant client who reports painless vaginal bleeding at 28 weeks gestation is diagnosed with
placenta previa. It was described as the placental edge reaches but not covers the internal os. The
nurse would suspect the client has which type of placenta previa?
A. Low lying placenta previa
B. Marginal placenta previa
C. Partial placenta previa
D. Total placenta previa

Rationale:
A marginal placenta previa is characterized by implantation of the placenta in the margin of the cervical os. A
low lying placenta is implanted in the lower uterine segment but doesn’t reach the cervical os. Partial placenta
previa is the partial occlusion of the cervical os by the placenta. The internal cervical os is completely covered
by the placenta in a total placenta previa.

(NCLEX-RN Questions & Answers made Incredibly Easy!, 6th ed. By Lisko, p. 1664)
65. The normal placenta weighs approximately 500 g and is 15 to 20 cm in diameter and 1.5 to 3.0 cm
thick. Its weight is approximately one sixth that of the fetus. The nurse reviews the client's medical
record. She found out that the client has placenta accreta. Based on her knowledge, she would
accurately describe this condition by stating:
A. The placenta invades the myometrium
B. The placenta covers the cervical os
C. The placenta penetrates the myometrium
D. The placenta attaches to the myometrium

Rationale:
Placenta accreta is the abnormal attachment of the placenta to the myometrium of the uterus. When the
placenta invades the myometrium, it is called placenta increta. When the placenta covers the cervical os, it is
called placenta previa. Placenta percreta occurs when the villi of the placenta penetrate the myometrium to the
serosa level.
(NCLEX-RN Questions & Answers made Incredibly Easy!, 6th ed. By Lisko, p. 1667)

Concept: Placenta previa


 Description and etiology
 linical manifestations
 Diagnostic tests
 Management

Abruptio placenta is the premature separation of a normally implanted placenta. The uterus in abruption is
characteristically hard, boardlike and painful. In labor, tetanic contractions are observed. The classic abruption
is central, concealed or covert type where bleeding accumulates behind the placenta and signs and symptoms
of hypovolemia are not proportional to the amount of external bleeding. If abruption is overt or external type,
the placental margins separate first and vaginal bleeding is old and dark red. Complications include
hemorrhage, disseminated intravascular coagulation, Couvelaire uterus and prematurity.

Situation 6: Shakira is a 30 year old multigravid client who is in her last trimester of pregnancy. She arrives at
the hospital with red vaginal bleeding. She states that the bleeding started suddenly.

66. The client on admission presents with rigid, board like tender abdomen, maternal shock and fetal
distress. Which assessment finding is considered a predisposing factor for the development of
abruption placenta?
A. Gestational diabetes
B. Hyperemesis gravidarum
C. Pregnancy induced hypertension
D. Oligohydramnios
Core competency: Safe and Quality Care– Analysis
Rationale:
The primary cause of premature separation of the placenta (abruption placenta) is unknown. But certain
predisposing factors have been identified as follows.
I. High parity
II. Advanced maternal age

III. Short umbilical cord


IV. Chronic hypertensive disease
V. Pregnancy-induced hypertension
VI. Direct trauma (automobile accident or intimate partner abuse)
VII. Vasoconstriction from cocaine or cigarette use
VIII. Thrombophillic conditions that lead to thrombosis such as autoimmune antibodies, protein C, and
factor V.
(PILLITERI 5TH ED., pg, 415-416)

67. If the client develops a complete abruption, the nursing care plan should include careful assessment for
signs and symptoms of which of the following?
A. Jaundice
B. Hypovolemic shock
C. Impending convulsion
D. Hypertension
Core competency: Safe and Quality Care– Analysis

Rationale:
Heavy bleeding usually accompanies premature separation of the placenta, external bleeding may
present if the placenta separates first at the edge and bloods escapes freely from the cervix. Signs of
complications may arise if continuous bleeding persist such as:
1) couvelaire uterus- infiltration of blood in the uterine musculature, forming a hard, boardlike uterus.
2) Shock- due to extensive blood loss
3) Disseminated intravascula coagulation
(PILLITERI 5TH ED., pg, 415-416)

Concept: Abruptio placenta


 Description and etiology
 Types
 Clinical manifestations
 Management
 Complications

68. Mrs. Cass K. Acero, has been involved in an automobile accident. She was 34 weeks pregnant. She
does not complain of any physical injury. Due to the nature of the accident, the nurse, would monitor
the client for which complication of pregnancy?
A. Placenta previa
B. Transverse fetal lie
C. Placental abruption
D. Severe preeclampsia

Rationale:
The fetus is well protected within the uterine body. The musculature of the uterus and the amniotic fluid provide
the baby with enough cushioning to withstand minor bumps and falls. A major automobile accident, however,
can cause anything from preterm premature rupture of the membranes, to a ruptured uterus, to placental
abruption. The nurse should especially monitor thefetal heart beat for any variations.
(Maternal and Newborn Success: A Course Review Applying Critical Thinking to Test Taking, by De Sevo, p.
242, 260)

69. Katie, a 10 week pregnant client, arrives at the emergency department with complaints of cramping,
abdominal pain, and mild vaginal bleeding. Pelvic examination shows a left palpable mass that is
tender. Culdocentesis shows blood in the cul-de-sac. The nurse suspects this client may have which
condition?
A. Abruptio placenta
B. Ectopic pregnancy
C. Hydatidiform mole
D. Pelvic inflammatory disease

Rationale:
Most ectopic pregnancies don’t appear as obvious life threatening medical emergencies. Ectopic pregnancies
must be considered in any sexually active woman of childbearing age who complains of menstrual irregularity,
cramping abdominal pain, and mild vaginal bleeding. The client with an ectopic pregnancy who is experiencing
blood loss will have blood in the cul-de-sac.

(NCLEX-RN Questions & Answers made Incredibly Easy!, 6th ed. By Lisko, p. 1168-1169)

Hyperemesis gravidarumis characterized by extreme nausea and vomiting during the first half of pregnancy
that is associated with dehydration, weight loss and electrolyte imbalances. High levels of hCG are associated
with severe nausea and vomiting. The fetus is at risk for abnormal development, IUGR, or death from lack of
nutrition, hypoxia, and maternal ketoacidosis.
Situation7. A 26 year old primigravid client visits her obstetrician for her first prenatal visit complaining of
severe nausea and frequent vomiting. The physician confirms that she is in the 14th week of pregnancy. The
nurse and client discuss general health needs during pregnancy.
70. The nurse anticipates a diagnostic test will be ordered to confirm that the condition is a plain
hyperemesis gravidarum and not due to other antepartal complications. Which response of the nurse is
most appropriate?
A. “You will be requested a biophysical profile.”
B. “Collect urine for Human chorionic gonadoptrophin examination.”
C. “Sign consent for amniocentesis.”
D. “Drink lots of fluid for abdominal ultrasound.”
Core competency: Safe and Quality Care – Analysis

Rationale:
Diagnostic test used for Hyperemesis gravidarum are
A. Labarotary analysis:
 Urinalysis for ketones and specific gravity: A sign of starvation, ketones may be harmful to fetal
development. High specific gravity occurs with volume depletion.
 Serum electrolytes (potassium or sodium) and ketones: Assess electrolyte status to evaluate for low
potassium or sodium, identify hyperchloremic metabolic alkalosis or acidosis, and evaluate renal
function and volume status.
 Liver enzymes and bilirubin: Elevated transaminase levels may occur in as many as 50% of patients
with hyperemesis gravidarum. Significantly elevated liver enzymes, however, may be a sign of another
underlying liver condition, such as hepatitis (viral, ischemic, autoimmune), or some other etiology of
liver injury.
 Urine culture: This may be indicated because urinary tract infection is common in pregnancy and can
be associated with nausea and vomiting.
 Hematocrit: This may be elevated because of volume contraction.
B. Imaging studies
˃ Obstetric ultrasonography is usually warranted in patients with HEG to evaluate for multiple gestations
or trophoblastic disease.
Website: http://emedicine.medscape.com/article/254751-workup#c7

71. The patient admitted was ordered IV fluid infusion at 125 ml/hr. The nurse’s primary nursing
responsibility would be?
A. Oxygen
B. Frequent rest periods
C. Input and output monitoring
D. Catheterization
Core competency: Safe and Quality Care - Analysis

Rationale:
Nursing measures:
˃ Intravenous fluid may be administered to increase hydration.
˃ An antiemetic (metoclopramide) to control vomiting
Note: careful measure of intake and out, including the amount of vomitus must be necessary.
˃ Assess the patient's abdomen every 2 hours or as her condition warrants, including size, contour, and
bowel sounds, and note pain, tenderness, and guarding. Also assess her vital signs. help diagnose
various disorders that cause vomiting, including liver disease, kidney infection, pancreatitis, GI
obstruction or lesions, drug toxicity, and intracranial lesions.
˃ Weigh the patient daily, if hospitalized, or at each visit. Using the same scale, weigh her at the same
time of day, making sure she's wearing the same type of clothing. Note patterns of weight gain.

Website:http://www.lavismedical.net/2014/09/hyperemesis-gravidarum-case-study-assessment-
nursing-care-plan.html
Book: (PILLITERI 5TH ED., pg, 320)

72. Aubrey, has been admitted with a diagnosis of hyperemesis gravidarum. Which of the following orders
written by the primary health care provider is highest priority for the nurse to complete?
A. Obtain complete blood count.
B. Start intravenous with multivitamins.
C. Check admission weight.
D. Obtain urine for urinalysis.

Rationale:
Clients who are vomiting repeatedly are energy depleted, vitamin depleted, and electrolyte depleted and are
often dehydrated. It is essential that the client receive her IV therapy as quickly as possible. The other orders
should be completed soon after the IV is started.

(Maternal and Newborn Success: A Course Review Applying Critical Thinking to TestTaking, by De Sevo, p.
255, 279)

Concept: Hyperemesis gravidarum


 Description and etiology
 Clinical manifestations
 Diagnostic tests
 Management
 Complications

Preeclampsia is a hypertensive disorder of pregnancy developing after 20 weeks gestation and is


characterized by triad signs of hypertension, proteinuria and edema. Assessment findings result from
generalized vasospasm and arteriolar vasoconstriction, which causes increased peripheral resistance,
decreased blood flow to the tissues, and hypertension.

Situation 8: A 22 year old primigravid client is in her 22nd week of pregnancy. The physician informed the client
that she has pregnancy induced hypertension. She is admitted to the hospital.
73. The nurse concludes that the client was diagnosed with pregnancy induced hypertension when the vital
signs taken today show that the blood pressure has increased during pregnancy from 100/60 to 130/80.
When assessing the client, the nurse should thoroughly explore which finding at each visit?
A. A decrease in urine protein level
B. An increase in urine output
C. A decrease in pulse rate
D. Any sudden weight gain
Core Competency: Safe and Quality Care– Analysis

Rationale:
A sudden weight gain may occur because of formation of edema. Edema formation is due to increased
tubular reabsorption of sodium causes by vasospasm in the kidney, edema is further increased because as
more protein is lost, the osmotic pressure of the circulating blood falls and fluid diffuses in from the circulatory
system into the denser interstitial spaces to equalize the pressure. Extreme edema can lead to cerebral and
pumonary edema and seizures.
Choice A, B, and C is not associated with PIH. Due to vasospasm in the kidney, blood resistance may
increases causing Increased permeability of the glomerular membrane, allowing the serum proteins albumin
and globulin to escape in the urine (proteinuria- presence protein in the urine); decreases glomerular filtration,
so there is lowered urine output and clearance of creatinine.
(PILLITERI 5TH ED., pg, 426-427)

74. A client with gestational hypertension is experiencing abdominal pain and vaginal bleeding. Which
assessment should the nurse perform first?
A. Assess fetal heart tones
B. Assess strength of contractions
C. Assess urinary output
D. Assess serum electrolytes

Rationale:
Since the findings suggest that the client is experiencing abruptio placenta, fetal heart tones should
immediately be assessed to determine fetal well-being. The other interventions should also be implemented,
but after the fetus is assessed.
(NCLEX-RN Questions & Answers made Incredibly Easy!, 6th ed. By Lisko, p. 1205-1206)

75. A nurse is monitoring a client who is receiving magnesium sulfate for preeclampsia and is assessing
the client every 30 minutes. Which of the following findings would indicate a need to immediately report
the findings?
A. Urinary output of 20 ml
B. Deep tendon reflexes of +2
C. Respirations of 10 breaths/minute
D. Fetal heart rate of 116 beats/min
Core Competency: Safe and Quality Care – Analysis

Rationale:
Nursing considerations when giving magnesium sulfate are as follows:
1) Infuse the drug slowly over 15-30 minutes
2) Assess respiratory rate, urine output, deep tendon reflexes, and clonus every hour.
3) Keep in mind that urine output should be over 30 ml/hour and respiratory rate over 12/min. Ankle
clonus(continued motion of the foot) should be minimal. Serum magnesium level should remain below
7.5 meq/L
4) Calcium gluconate solutions should be kept ready if signs and symptoms of magnesium sulfate toxicity.
5) Monitor for CNS depression
(PILLITERI 5TH ED., pg, 430)

Content; Preeclampsia
 Description
 Clinical manifestations
 Management

Eclampsia is used to describe preeclampsia that has progressed to include maternal tonic-clonic seizures or
coma. Complications include cerebral edema, stroke, abruption placenta and fetal death. There is a high risk of
developing HELLP syndromes which stands for hemolysis, elevated liver enzymes, and low platelets.
Situation 9: A pregnant client in her last trimester has been admitted to the hospital. Her initial admitting vital
signs are blood pressure 160/90; pulse 88; respirations 24 and temperature 98 F.
76. The client complains of epigastric pain and headache. What should the nurse do INITIALLY?
A. Insert and indwelling catheter
B. Give Maalox 30 cc now
C. Contact the doctor STAT with findings
D. Provide supportive care for impending convulsions
Core Competency: Safe and Quality Care – Analysis

Rationale:
The most severe classification of PIH is Eclampsia. A woman has passed into the stage when cerebral
edema is so acute that a seizure or coma occurs. So the goal treatment is Seizure precaution.
(PILLITERI 5TH ED., pg, 428- 431)

77. The physician ordered magnesium sulfate deep IM. While the client is receiving magnesium sulfate, the
nurse routinely assesses the client’s vital signs and notes the following: BP 160/90, and blurring of
vision. In caring for the client the nurse should:
A. Encourage her to drink clear fluids
B. Protect her against strenuous stimuli
C. Isolate her in a dark room
D. Maintain her in a supine position
Core Competency: Safe and Quality Care – Analysis

Rationale:
- Most evident symptoms of magnesium sulfate overdose are:
 Decreased urine output
 Depressed respirations
 Reduced consciousness
 Decreased deep tendon reflexes
- Action of magnesium sulfate in patient with PIH, muscle relaxant, CNS depressants which lessens
the possibility of seizures.
(PILLITERI 5TH ED., pg, 430)

78. Nurse Andrew is caring for a client receiving magnesium sulfate intravenously. The nurse understands
that based on the client's condition, which of the following medications should be present in the
bedside?
A. Hydralazine
B. Naloxone
C. Calcium gluconate
D. RhoGAM

Rationale:
Calcium gluconate is the antidote for magnesium toxicity. Ten milliliters of 10% calcium gluconate is given I.V.
push over 3 to 5 minutes. Hydralazine is given for sustained elevated blood pressures in preeclamptic clients.
Naloxone s used to correct narcotic toxicity. RhoGAM is given for RH incompatibility
(NCLEX-RN Questions & Answers made Incredibly Easy!, 6th ed. By Lisko, p. 1181)

79. A client is afraid of another convulsion and asks when the likelihood of convulsions will end. The nurse
replies that the danger of a convulsion in a woman with eclampsia ends:
A. After labor begins
B. After delivery occurs
C. 24 hours postpartum
D. 48 hours postpartum
Core Competency: Safe and Quality Care – Application

Rationale:
Degeneration of a woman’s condition from severe preeclampsia to eclampsia occurs when cerebral
irritation from increasing cerebral edema becomes so acute that a seizure occurs. This is usually happens late
in pregnancy but happen up to 48 hours after childbirth.
(PILLITERI 5TH ED., pg, 431)

Concept: Eclampsia
 Description
 Clinical manifestations
 Management
 Complications

Rheumatic heart disease and congenital heart defect would be the leading conditions that can result in a
cardiac disease complicating pregnancy. The major threat imposed by pregnancy in patients with heart
disease is increased blood volume and potential congestive heart failure. Signs and symptoms include cough,
dyspnea, edema, and arrhythmia. Therapeutic classification of heart disease includes: Class 1: no limitation of
physical activity; Class II: slight limitation of physical activity; Class III: moderate to marked limitation of
physical activity; Class IV: marked limitation of physical activity.Classes I and II can be allowed pregnancy and
delivery whereas Classes III and IV are poor candidates.

Situation 10: A 25 year old woman is four months pregnant. She had rheumatic fever at age 15 and developed
a systolic murmur. She reports exertional dyspnea.
80. The client has been instructed on home management. Which instruction should the nurse give her?
A. “Try to keep as active as possible, but eliminate any activity that you find tiring.”
B. “Carry on all your usual activities, but learn to work at a slower pace.”
C. “Avoid heavy housework, shopping, stair climbing, and all unnecessary physical effort.”
D. “Get someone to do your housework, and stay in bed or in a wheelchair.”
Core Competency: Health Education– Application
Rationale:
Therapeutic classification of heart disease by New York State Heart Association includes:

Class I – Uncompromised: women have no limitation of physical activity. Ordinary physical activity causes no
discomfort. They have no symptoms of cardiac insufficiency and no anginal pain.

Class II – Slightly compromised: women have slight limitation of physical activity. Ordinary physical activity
causes excessive fatigue, palpitation, and dyspnea or anginal pain.

Class III – Markedly compromised: women have moderate to marked limitation of physical activity. During less
than ordinary activity, they experience fatigue, palpitations, dyspnea, or anginal pain.

Class IV – Severely compromised: women are unable to carry out any physical activity without discomfort.
Even at rest they experience symptoms of cardiac insufficiency or anginal pain.

Answer is letter C because the woman experiencing discomfort during doing less than ordinary physical
activity. While choice A is on the Class II, choice B is on Class I and choice D on class IV.
(PILLITERI 5TH ED., pg, 354)
81. The client is concerned about the delivery of her baby and asks what to expect. Which instruction
should the nurse give to her during the second stage of labor?
A. Breathe shallowly and rapidly
B. Sit on the side of the bed
C. Sleep between contractions
D. Avoid prolonged bearing down
Core competency: Safe and Quality Care– Application
Rationale:
Pregnant women with heart disease should not push with contraction or bear down; because pushing
may requires more effort than they should expend. Instead, a epidural anesthetic is used, low forceps
or a vacuum extractor can be use for birth. Other care measures:
1) Monitor fetal heart rate and uterine contractions during labor in all women with hear disease.

2) Assess woman’s blood pressure, pulse and respirations frequently. A rapidly increasing pulse
rate (more than 100 bpm) is an indication that a heart is pumping ineffectively and has
increased its rate in an effort to compensate.
3) Advise woman to assume a side – lying position to reduce possibility of supine hypotension
syndrome.
4) Semi –fowlers position, may be necessary if she has an pulmonary edema
(PILLITERI 5TH ED., pg, 359)

82. Nurse Anne is providing instructions to a maternity client with a history of cardiac disease regarding
appropriate dietary measures. Which statement, if made by the client, indicates an understanding of the
information provided by the nurse?
A. “I should increase my sodium intake during pregnancy.”
B. “I should lower my blood volume by limiting my fluids.”
C. “I should maintain a low-calorie diet to prevent any weight gain.”
D. “I should drink adequate fluids and increase my intake of high-fiber foods.”

Rationale:
Valsalva maneuver should be avoided in clients with cardiac disease because it can cause blood to rush to the
heart and overload the cardiac system. Constipation can cause the client to use Valsalva maneuver. High-fiber
foods are important. A low-calorie diet is not recommended during pregnancy and could be harmful to the
fetus. Diets low in fluid can cause a decrease in blood volume, which could deprive the fetus of nutrients, so
adequate fluid intake and high-fiber foods are important. Sodium should be restricted as prescribed by the
physician because excess sodium would cause an overload to the circulating blood volume and contribute to
cardiac complications.
(Saunders Comprehensive Review for the NCLEX-RN Examination, 5th ed. By Silvestri, p 294,297)

Concept: Cardiac disease


 Description and etiology
 Types
 Clinical manifestations
 Management

Diabetes mellitus is a chronic hereditary disease characterized by marked hyperglycemia due to abnormalities
in CHO, fats and protein metabolism. The classic triad signs of diabetes mellitus are excessive appetite,
excessive urination and excessive thirst. Diabetes mellitus during pregnancy may be: pregestational (type 1
and type 2) and gestational. Hazards of diabetes include history of anomalies, stillbirths and fetal deaths,
excessively large and cesarean birth.

Situation 11: A nursing board reviewer is Class B, insulin dependent diabetic, and is receiving prenatal care in
a high risk obstetric clinic. Her diabetes has been well controlled since her initial diagnosis.
83. The nurse implements a teaching plan for a pregnant client who was diagnosed with diabetes mellitus.
The nurse understands that the diabetic mother’s metabolism is significantly altered during pregnancy
as a result of:
A. The lower renal threshold for glucose
B. The increased effect of insulin during pregnancy
C. An increase in the glucose tolerance level of the blood
D. The effect of hormones produced in pregnancy on carbohydrate and lipid metabolism
Core competency: Health Education– Analysis

Rationale:
It is unknown whether gestational diabetes results from inadequate insulin response to carbohydrate or
excessive resistance to insulin. (PILLITERI 5TH ED., pg, 380)

84. The client is in her last trimester of pregnancy and her diabetes has been well controlled. She tells the
nurse that she is excited but also scared that something could be wrong with her baby because of her
diabetes. Which response of the nurse is most appropriate?
A. “Your baby may be small but otherwise healthy.”
B. “Your baby may be large and initially will need blood glucose monitoring.”
C. “Your baby will be diabetic.”
D. “Your baby will have a minor birth defect.”
Core competency: Health Education– Application

Rationale:

All women with diabetes need to do blood glucose monitoring to determine whether hypoglycemia or
hyperglycemia exists. Also a woman with diabetes, the weight of the infant is directly correlated with what she
gains during pregnancy. The more she gains and not maintaining the more her baby becomes larger.
(PILLITERI 5TH ED., pg, 380-382)

85. A student-nurse, together with Nurse Danni implements a teaching plan for a pregnant client who is
newly diagnosed with gestational diabetes mellitus. Which statement made by the client indicates a
need for further teaching?
A. “I should stay on the diabetic diet.”
B. “I should perform glucose monitoring at home.”
C. “I should avoid exercise because of the negative effects on insulin production.”
D. “I should be aware of any infections and report signs of infection immediately to my health care
provider.”

Rationale:
Exercise is safe for a client with gestational diabetes mellitus and is helpful in lowering the blood glucose level.
Dietary modifications are the mainstay of treatment, and the client is placed on a standard diabetic diet. Many
clients are taught to perform blood glucose monitoring. If the client is not performing the blood glucose
monitoring at home, it is performed at the clinic or health care provider’s office. Signs of infection need to be
reported to the health care provider.
(Saunders Comprehensive Review for the NCLEX-RN Examination, 5th ed. By Silvestri, p 293,295)

Concept: Diabetes Mellitus


 Description and etiology
 Types
 Clinical manifestations
 Management
 Health teachings

Iron deficiency anemia is the most common medical disorder of pregnancy characterized by decrease in
oxygen-carrying capacity of the blood. Client is pale, tired, short of breath and dizzy. Nursing interventions
would include encouraging intake of food with high iron content, monitoring iron supplementation and teaching
sequelae of iron ingestion.
86. A nurse discusses high-risk complications with a group of women at a prenatal clinic. Margaute, age
22, primipara during her 2nd trimester has asked the nurse regarding her hemoglobin result of 9 g/dL.
and has been prescribed 300 mg of ferrous sulfate daily for her pregnancy-related anemia. To assess
compliance, the nurse should do which of the following?
A. Assess hemoglobin and hematocrit level
B. Do a Guaiac stool test
C. Weigh Margaute
D. Ask Margaute if she is taking the medication.
Core competency: Safe and Quality Care – Analysis

Rationale:
Normal Hemoglobin in pregnant mother:
Normal hematocrit level in pregnant mother:
In a patient with Iron deficiency anemia the hemoglobin and hematocrit level are decreased
(hemoglobin - <12m g/dl ; hematocrit - <33 %). The best intervention is giving prenatal vitamins containing an
iron supplements in the form of Ferrous sulfate. Therefore, when a woman taking the prescribed iron
supplement, new red blood cells should begin to increase or reticulocyte counts should rise by 2 weeks.
(PILLITERI 5TH ED., pg, 362)
Concept: Iron deficiency anemia
 Description
 Clinical manifestations
 Diagnostic test
 Management

87. Sickle cell anemia is a recessively inherited hemolytic anemia caused by an abnormal amino acid in the
beta chain of hemoglobin. A pregnant client with sickle cell anemia is at an increased risk for having a
sickle cell crisis during pregnancy. The nurse anticipates that aggressive management of a sickle cell
crisis would include which treatment?
A. Antihypertensive agents
B. Diuretic agents
C. Intravenous fluids
D. Acetaminophen for pain

Rationale:
A sickle cell crisis during pregnancy is usually managed by exchange transfusion, oxygen, and I.V. fluids.
Antihypertensive drugs usually aren’t necessary. Diuretics wouldn't be used unless fluid overload resulted. The
client usually needs a stronger analgesic than acetaminophen to control the pain of a crisis.
(NCLEX-RN Questions & Answers made Incredibly Easy!, 6th ed. By Lisko, p. 1177)

Urinary tract infection affects 10% of all pregnant women. E. coli is the usual cause. UTI are manifested as
infections involving the bladder, kidney and urethra. Predisposing factors include normal partum diuresis,
increased bladder capacity, decreased bladder sensitivity, and contamination. The presence of nitrites, white
blood cells, and bacteria are all indicative of a urinary tract infection. If untreated with antibiotics, it may
predispose to premature labor.
88. The nurse conducts a prenatal class on high risk factors during pregnancy. Several participants in the
prenatal class complain of frequent urination. The nurse correctly explains to the group that the most
commonly assessed findings in cystitis are which of the following?
A. Frequency, urgency, hematuria, nausea, chills, and flank pain
B. Nocturia, frequency, urgency, dysuria, hematuria, fever, and suprapubic pain
C. Dehydration, hypertension, dysuria, suprapubic pain, chills and fever
D. High fever, chills, flank pain, nausea, vomiting, dysuria and frequency
Core competency: Safe and Quality Care– Analysis

Rationale:
Sign and symptoms of UTI are as follows
a. Cystitis – suprapubic pain, hypertension, dysuria, hematuria, fever, urgency and nocturnal frequency
b. Pyelonephritis – flank pain (pain in the lumbar region that radiates downward, nausea and vomiting,
pain and frequency in urination, fever, hematuria and chills.
(PILLITERI 5TH ED., pg, 430)

Concept: Urinary tract infections


 Description and etiology
 Clinical manifestations
 Diagnostic criteria\
 Management

Pelvic inflammatory disease (PID) is infection of the pelvic organs: the uterus, fallopian tubes, ovaries, and
their supporting structures. The infection can extend to cause pelvic peritonitis.
89. Nurse Gina is doing a health teaching about the Pelvic Inflammatory Disease. She is incorrect if she
states which of the following?
A. Symptoms of PID include heavy vaginal discharge
B. Early childbearing may be suggested to a woman with history or current PID
C. The couple may have coitus during menstruation where protective mechanisms are adequate to
protect the woman
D. Diagnosis may be aided by laparoscopy or ultrasound

Rationale:
Assessment:
- severe pain in the lower abdomen
- heavy, purulent discharge
- may develop fever
- leukocytosis and elevated erythrocyte sedimentation rate
- severe pain during pelvic examination

Diagnosis can be aided by ultrasound or laparoscopy.


Management:
- Administration of analgesia for comfort plus specific broad-spectrum antibiotics
- avoid coitus with an infected partner,
-avoid coitus during menstruation, when their protective mechanisms are lowest
- early childbearing may be recommended if they plan to have children, because extensive tubal scarring
could impair fertility
(Maternal and Childhealth Nursing, 6th ed. By Pilliterri, p. 1399-1400)

In Tuberculosis, lung tissue is invaded by Mycobacterium tuberculosis. Fibrosis, calcification, and a final ring of
collagenous scar tissue develop, effectively sealing off the organisms from the body and any further invasion or
spread.
90. Leslie, 15 weeks pregnant, is screened for tuberculosis during her first prenatal visit in the health
center. A Purified Protein Derivative PPD test was done. The result of the test will be positive if which of
the following occurs?
A. A flat circumscribed area under 10 mm in diameter appears in 6 to 12 hours
B. A flat circumscribed area over 10 mm in diameter appears in 48 to 72 hours
C. An indurated wheal under 10 mm in diameter appears in 6 to 12 hours
D. An indurated wheal over 10 mm in diameter appears in 48 to 72 hours
Rationale:
A positive PPD result would be an indurated wheal over 10 mm in diameter that appears in 48 to 72 hours. The
area must be a raised wheal not a flat circumscribed area.
(NCLEX-RN Questions & Answers made Incredibly Easy!, 6th ed. By Lisko, p. 1181-1182)

91. Leslie, also reports loss of appetite, weight loss, and fatigue. The nurse provides instructions to the
client regarding therapeutic management of the tuberculosis and tells the client that:
A. Therapeutic abortion is required.
B. She will have to stay at home until treatment is completed.
C. Medication will not be started until after delivery of the fetus.
D. Isoniazid (INH) plus Rifampin (Rifadin) will be required for 9 months.
Rationale:
More than one medication may be used to prevent the growth of resistant organisms in a pregnant client with
tuberculosis. Treatment must continue for a prolonged period. The preferred treatment for the pregnant client is
isoniazid (INH) plus rifampin daily for 9 months. Ethambutol is added initially if medication resistance is
suspected. Pyridoxine (vitamin B6) often is administered with INH to prevent fetal neurotoxicity. The client does
not need to stay at home during treatment, and therapeutic abortion is not required.
(Saunders Comprehensive Review for the NCLEX-RN Examination, 5th ed. By Silvestri, p 294, 296)
Several diseases that are commonly known to cross the placenta and cause fetal harm are tested for and are
described collectively under the umbrella term TORCH. These Teratogenic maternal infections can involve
either sexually transmitted or systemic infections. These organisms can be viral, bacterial, or protozoan.

92. Mr. Tim Ang is very upset when he heard that during pregnancy, her wife acquired one of the maternal
infections under TORCH. He says, "This means the baby has HIV!" The nurse replies that the H in
TORCH represents which of the following conditions?
A. Hepatitis B virus
B. Herpes simplex virus
C. Hemophilia
D. Human immunodeficiency virus

Rationale:
TORCH represents the following maternal infections: Toxoplasmosis; Other Infections such as gonorrhea,
syphilis, varicella, hepatitis and HIV; Rubella, Cytomegalovirus; and Herpes Simplex
(NCLEX-RN Questions & Answers made Incredibly Easy!, 6th ed. By Lisko, p. 1277-1278)
93. A pregnant client with genital herpes consulted the health care facility. The most important instruction
that the nurse must provide about the measures to protect the fetus is which of the following?
A. Total abstinence from sexual intercourse is necessary during the entire pregnancy.
B. Sitz baths need to be taken every 4 hours while awake if vaginal lesions are present.
C. Penicillin is the drug of choice and could be given during the pregnancy.
D. A cesarean section will be necessary if vaginal lesions are present at the time of labor.

Rationale:
For women with active lesions, either recurrent or primary at the time of labor, delivery should be by cesarean
section to prevent the fetus from being in contact with the genital herpes.
Acyclovir (not penicillin)can be given to woman with herpes simplex virus
Clients should be advised to abstain from sexual contact while the lesions are present. If this is an initial
infection, clients should continue to abstain until they become culture-negative because prolonged viral
shedding may occur in such cases. Keeping the genital area clean and dry promotes healing.
(Saunders Comprehensive Review for the NCLEX-RN Examination, 5th ed. By Silvestri, p 264-265)
94. A 5 week pregnant client is diagnosed to have Syphilis. She is worried that her baby might be affected
because of its teratogenic effects. Which statement by the nurse would be the most inaccurate?
A. The drug of choice for syphilis is Penicillin.
B. Treponema pallidum is its causative agent
C. In the first trimester of pregnancy, the fetus will not be affected because of the protective function of the
syncitiotrophoblast
D. If it is not treated, hearing impairment, osteochondritis and fetal death may occur
Rationale:
Syphilis, a sexually transmitted infection. Early in pregnancy, when the cytotrophoblast (not syncitiotrophoblast)
layer of the chorionic villi is still intact, the causative spirochete of syphilis, Treponema pallidum, cannot cross
the placenta and damage the fetus. When this layer atrophies at about the 16th to 18th week of pregnancy,
however, the spirochete then can cross and cause extensive damage. If syphilis is detected and treated with
an antibiotic such as benzathine penicillin in the first trimester, a fetus is rarely affected. If left untreated beyond
the 18th week of gestation, hearing impairment, cognitive challenge, osteochondritis, and fetal death are
possible.
(Maternal and Childhealth Nursing, 6th ed. By Pilliterri, p. 290)
95. Sexually transmitted infections may be teratogenic. All of them might be a factor in causing preterm
labor. Celia, is pregnant with her second child is diagnosed to have Trichomoniasis. During
assessment, how would Nurse Lydia describe the client's vaginal discharge?
A. frothy white or grayish-green
B. thick, cream cheese–like discharge
C. milk-white to gray and has a fishlike odor
D. slight yellowish vaginal discharge

Rationale:
Vaginal Discharge
Trichomoniasis: frothy white or grayish-green
Candidiasis: thick, cream cheese–like discharge
Bacterial vaginosis: milk-white to gray and has a fishlike odor
Gonorrhea: slight yellowish vaginal discharge
(Maternal and Childhealth Nursing, 6th ed. By Pilliterri, p. 1403-1406)
Infertility is the inability to conceive after at least 1 year of unprotected sexual relations. It is frequently caused
by multiple, rather than a single factor. Factors contributing to female infertility include the following: vaginal
problems, cervical problems, uterine problems, tubal problems and ovarian problems. Male factors would
include congenital factors, ejaculation problems, sperm abnormalities, testicular abnormalities, coital difficulties
and drugs. Evaluation of infertility must begin with a complete health history and physical examination of both
partners, and basic laboratory tests, including complete blood count, thyroid functions tests and urinalysis. If
these are negative, an infertility work-up consisting of more invasive and intensive diagnostic studies begin.
Education regarding cervical mucus changes can help the woman plan sexual intercourse to increase chances
of pregnancy.

Situation 15. A client married for two years comes to the infertility clinic for her gynecological examination. The
client is concerned and can’t understand why she hasn’t gotten pregnant.

96. The client has been diagnosed with Trichomonas vaginitis. The nurse explains during client teaching
that this infection can affect fertility by:
A. Utilizing the glycogen in vaginal secretions, leaving no nutrition for the spermatozoa
B. Creating a blockage of the fallopian tubes that prohibits spermatozoa from reaching an ovum.
C. Decreasing the pH of the vaginal secretions, thereby destroying most spermatozoa
D. Increasing the temperature inside the vagina, which decreases the motility of spermatozoa
Core Competency: Health Education–Application

Rationale:
In a patient with vaginal infection can cause the pH of the vaginal secretions to become acidotic,
limiting or destroying the motility of the spermatozoa. Some women appear to have sperm – immobilizing or
sperm – agglutinating antibodies in their blood plasma that act to destroy sperm cells in the vagina or cervix.
(PILLITERI 5TH ED., pg, 139)

97. The client is experiencing an inability to become pregnant after she has had one full term pregnancy.
The nurse should develop a plan of care for which health problem?
A. Primary infertility
B. Secondary infertility
C. Unexplained infertility
D. Combined factor infertility
Core Competency: Health Education–Analysis
Rationale:
 Primary infertility – there have been no previous conception or pregnancy
 Secondary infertility – there has been a previous viable pregnancy but the couple is unable to conceive
at present.
 Unexplained infertility – is infertility that is idiopathic in the sense that its cause remains unknown. This
catch-all term is used when doctors can't find a cause for infertility after a full series of tests and
assessments. Some experts think being significantly overweight or underweight, exercising excessively
and even environmental toxins may be contributing factors.
 Combined factor infertility – the term used to describe couples who have both male and female infertility
problems, or when one partner has more than one fertility problem.

Website:https://en.wikipedia.org/wiki/Unexplained_infertility, ;http://www.babycentre.co.uk/a6020/major-
causes-of-infertility-chart#ixzz3i6nLIKBy
Book: (PILLITERI 5TH ED., pg, 136)

Management of infertility involves treating the underlying causes. Medications may be used. Antibiotic therapy
treats infections. Testosterone treats oligospermia. Estrogen therapy increases the abundance of cervical
mucus and enhances spinnbarkeit. Clomiphine citrate (Clomid) is also used.

98. A married client has come to the infertility clinic. Which of the following information does the nurse need
to gather before scheduling a client for endometrial biopsy?
A. Usual length of menstrual cycle
B. Blood type and Rh factor
C. Presence of any metal implants
D. Last type of birth control use

Core competency: Health Education – Application

Rationale:
Uterine endometrial biopsy may be used as a test for ovulation or to reveal an endometrial problem
such as a luteal phase defect. The biopsy is usually done 2 or 3 days before the expected menstrual flow (day
25 or 26 of a typical 28 day menstrual cycle. (PILLITERI 5TH ED., pg, 145)
99. The nurse would understand that in vitro fertilization and embryo transfer (IVF-ET) would be a viable
reproductive alternative for which of the following?
A. A couple where the woman who is unable to carry a fetus to viability
B. A woman with damaged fallopian tubes but still ovulates regularly
C. A couple where the male partner is fertile and the woman is infertile
D. A woman unable to produce normal mature follicles with a fertile male partner
Core Competency: Health Education– Analysis

Rationale:
In vitro fertilization – most often used for couples who have not been able to conceive because the
woman has blocked or damaged fallopian tubes. It is also used when the man has oligospermia or low sperm
count.
Indications:
A Couples with unexplained infertility
B Couples with an absent cervical mucus prevents sperm from traveling to or entering the
cervix.
C Ovulatory disorders
D Pelvic inflammatory disease with severe adhesion
E Premature Menopause
F Severe endometriosis
(PILLITERI 5TH ED., pg, 148)

100. A couple has been unable to conceive. The client couple is planning to undergo in vitro
fertilization (IVF) and gamete intrafallopian tube transfer (GIFT). Which of the following statements
indicates to the nurse the need for additional information?
A. “I will be given medication injection to cause my ovaries to ripen more than one egg.”
B. “My husband will be asked to produce his sperms on the day my eggs are retrieved.”
C. “I will be confined in the hospital overnight for the procedure.”
D. “I can expect to have some discomfort during the procedure.”

Core competency: Health Education – Analysis

Rationale:
In vitro fertilization – The procedure takes about 20 to 30 minutes and is performed on an outpatient
basis. The woman is generally advised to rest on the day of the procedure. Some light spotting and/or
mild cramping may occur after the retrieval procedure.
Gamete intrafallopian tube transfer - woman and her partner spend about half a day at the doctor's
office or clinic, having their eggs and sperm retrieved and surgically transferred to the fallopian tube.

Website: http://www.babycenter.com/0_fertility-treatment-gamete-intrafallopian-transfer-gift_4095.bc
website:http://www.emedicinehealth.com/in_vitro_fertilization/page4_em.htm
Concept: Infertility
 Description and etiology
 Factors contributing to infertility
 Diagnostic evaluation
 Management
 Alternatives for infertile couples

You might also like